Will These Integrals Converge or Diverge as x Approaches Infinity?

Click For Summary
The discussion focuses on determining the convergence or divergence of two integrals as x approaches infinity. The first integral, ∫ from 1 to ∞ of (x^2)/(x^4 + 1) is compared to known divergent integrals, leading to confusion about using a smaller integral to infer divergence. The second integral, ∫ from 1 to ∞ of (x^2 - 6x + 1)/(x^2 + 4), is analyzed by breaking it down, revealing that it diverges because it is greater than the integral of 1, which is known to diverge. The participants clarify their understanding of the behavior of these integrals, ultimately confirming the divergence of the second integral. The discussion highlights the importance of comparing integrals and understanding their limits as x approaches infinity.
Jacobpm64
Messages
235
Reaction score
0

Homework Statement


Use the box and the behavior of rational and exponential functions as x \rightarrow \infty to predict whether the integrals converge or diverge.

Here is the box:
\int^\infty_1 \frac{1}{x^p} dx converges for p > 1 and diverges for p < 1.

\int^1_0 \frac{1}{x^p} dx converges for p < 1 and diverges for p > 1.

\int^\infty_0 e^{-ax} dx converges for a > 0.

Problem 1:
\int^\infty_1 \frac{x^2}{x^4 + 1} dx

Problem 2:
\int^\infty_1 \frac{x^2 - 6x + 1}{x^2 + 4} dx

Homework Equations


The ones in the box above.

The Attempt at a Solution


Problem 1:
I know that this integral is less than \int^\infty_1 \frac{1}{x} dx. I also know that \int^\infty_1 \frac{1}{x} dx diverges. This does not help me though because I can not use a diverging integral to say that a smaller integral is also diverging. This is where I'm confused.

Problem 2:
I know that this integral is less than \int^\infty_1 1 dx I also know that \int^\infty_1 1 dx diverges. This does not help me though because I can not use a diverging integral to say that a smaller integral is also diverging. So, I'm confused at the same place on this problem.
 
Physics news on Phys.org
For problem 1, isn't (x^2)/((x^4)+1) always less than 1/(x^2) for values of x > 1?

For problem 2, what does the integrand approach as x approaches infinity?
 
\frac{x^2}{x^4+1}&lt; \frac{x^2}{x^4}= \frac{1}{x^2}

For the second one, divide first:
\frac{x^2+ 6x+ 1}{x^2+ 4}= 1+ \frac{6x- 3}{x^2+ 4}[/itex]<br /> What does that first &quot;1&quot; tell you?
 
Oops, I copied down the wrong first problem.

The second one was correct though.

I understand the first one, thanks for that.

I still don't understand what you mean by what does the first "1" tell me.

Wait, does that mean.. that my integral is bigger than the integral of 1, so if the integral of 1 diverges, I can say that my integral also diverges!

woot!

Thanks!

I'm going to post the correct problem for #1, in a new post. Thanks.
 
Question: A clock's minute hand has length 4 and its hour hand has length 3. What is the distance between the tips at the moment when it is increasing most rapidly?(Putnam Exam Question) Answer: Making assumption that both the hands moves at constant angular velocities, the answer is ## \sqrt{7} .## But don't you think this assumption is somewhat doubtful and wrong?

Similar threads

  • · Replies 3 ·
Replies
3
Views
1K
  • · Replies 105 ·
4
Replies
105
Views
6K
Replies
3
Views
2K
  • · Replies 5 ·
Replies
5
Views
2K
  • · Replies 6 ·
Replies
6
Views
2K
  • · Replies 22 ·
Replies
22
Views
3K
  • · Replies 5 ·
Replies
5
Views
2K
  • · Replies 2 ·
Replies
2
Views
2K
Replies
7
Views
2K
  • · Replies 6 ·
Replies
6
Views
2K